Mom, Baby Exam 1

Ace your homework & exams now with Quizwiz!

A newly pregnant patient visits her provider's office for the first prenatal appointment. To estimate accurate weight gain throughout the pregnancy, the nurse will be evaluating the appropriateness of weight for height using the body mass index (BMI). The patient weighs 51 kg and is 1.57 m tall. The BMI is: ___________________

ANS: 20.7 BMI = weight divided by height squared. BMI = 51 kg/(1.57 m)2, or 20.7. Prepregnant BMI can be classified into the following categories: <18.5, underweight or low; 18.5 to 24.9, normal; 25 to 29.9 overweight or high; and >30, obese.

A woman is 6 weeks pregnant. She had a previous spontaneous abortion at 14 weeks of gestation and a pregnancy that ended at 38 weeks with the birth of a stillborn girl. What is her gravidity and parity using the GTPAL system? ___________________

ANS: 3-1-0-1-0 The correct calculation of this woman's gravidity and parity is 3-1-0-1-0. Using the GPTAL system, this patient's gravidity and parity information is calculated as follows: G: Total number of times the woman has been pregnant (she is pregnant for the third time). T: Number of pregnancies carried to term (she has had only one pregnancy that resulted in a fetus at term). P: Number of pregnancies that resulted in a preterm birth (none). A: Abortions or miscarriages before the period of viability (she has had one). L: Number of children born who are currently living (she has no living children).

A woman arrives at the clinic for a pregnancy test. The first day of her last menstrual period (LMP) was September 10, 2013. Her expected date of birth (EDB) would be? ___________________

ANS: June 17, 2014 Using Nägele's rule, June 17, 2014, is the correct EDB. The EDB is calculated by subtracting 3 months from the first day of the LMP and adding 7 days + 1 year to the day of the LMP. Therefore, with an LMP of September 10, 2013: September 10, 2013 - 3 months = June 10, 2013 + 7 days = June 17, 2013 + 1 year = June 17, 2014

A woman asks the nurse, "What protects my baby's umbilical cord from being squashed while the baby's inside of me?" The nurse's best response is: a. "Your baby's umbilical cord is surrounded by connective tissue called Wharton jelly, which prevents compression of the blood vessels and ensures continued nourishment of your baby." b. "Your baby's umbilical floats around in blood anyway." c. "You don't need to worry about things like that." d. "The umbilical cord is a group of blood vessels that are very well protected by the placenta."

ANS: A "Your baby's umbilical cord is surrounded by connective tissue called Wharton jelly, which prevents compression of the blood vessels and ensures continued nourishment of your baby" is the most appropriate response. "Your baby's umbilical floats around in blood anyway" is inaccurate. "You don't need to worry about things like that" is an inappropriate response. It negates the patient's need for teaching and discounts her feelings. The placenta does not protect the umbilical cord. The cord is protected by the surrounding Wharton jelly.

As relates to the father's acceptance of the pregnancy and preparation for childbirth, the maternity nurse should know that: a. the father goes through three phases of acceptance of his own. b. the father's attachment to the fetus cannot be as strong as that of the mother because it does not start until after birth. c. in the last 2 months of pregnancy, most expectant fathers suddenly get very protective of their established lifestyle and resist making changes to the home. d. typically men remain ambivalent about fatherhood right up to the birth of their child.

ANS: A A father typically goes through three phases of development to reach acceptance of fatherhood: the announcement phase, the moratorium phase, and the focusing phase. The father-child attachment can be as strong as the mother-child relationship and can also begin during pregnancy. In the last 2 months of pregnancy, many expectant fathers work hard to improve the environment of the home for the child. Typically, the expectant father's ambivalence ends by the first trimester, and he progresses to adjusting to the reality of the situation and then to focusing on his role.

A woman who has completed one pregnancy with a fetus (or fetuses) reaching the stage of fetal viability is called a: a. primipara. b. primigravida. c. multipara. d. nulligravida.

ANS: A A primipara is a woman who has completed one pregnancy with a viable fetus. To remember terms, keep in mind: gravida is a pregnant woman; para comes from parity, meaning a viable fetus; primi means first; multi means many; and null means none. A primigravida is a woman pregnant for the first time. A multipara is a woman who has completed two or more pregnancies with a viable fetus. A nulligravida is a woman who has never been pregnant.

Identify the goal of a patient with the following nursing diagnosis: Imbalanced Nutrition: Less Than Body Requirements related to diet choices inadequate to meet nutrient requirements of pregnancy. a. Gain a total of 30 lbs. b. Take daily supplements consistently. c. Decrease intake of snack foods. d. Increase intake of complex carbohydrates.

ANS: A A weight gain of 30 lbs is one indication that the patient has gained a sufficient amount for the nutritional needs of pregnancy. A daily supplement is not the best goal for this patient. It does not meet the basic need of proper nutrition during pregnancy. Decreasing snack foods may be a problem and should be assessed; however, assessing weight gain is the best method of monitoring nutritional intake for this pregnant patient. Increasing the intake of complex carbohydrates is important for this patient, but monitoring the weight gain should be the end goal.

A pregnant patient would like to know a good food source of calcium other than dairy products. Your best answer is: a. legumes. b. yellow vegetables. c. lean meat. d. whole grains.

ANS: A Although dairy products contain the greatest amount of calcium, it is also found in legumes, nuts, dried fruits, and some dark green leafy vegetables. Yellow vegetables are rich in vitamin A. Lean meats are rich in protein and phosphorus. Whole grains are rich in zinc and magnesium.

A woman at 10 weeks of gestation who is seen in the prenatal clinic with presumptive signs and symptoms of pregnancy likely will have: a. amenorrhea. b. positive pregnancy test. c. Chadwick's sign. d. Hegar's sign.

ANS: A Amenorrhea is a presumptive sign of pregnancy. Presumptive signs of pregnancy are felt by the woman. A positive pregnancy test, the presence of Chadwick's sign, and the presence of Hegar's sign are all probable signs of pregnancy.

A maternity nurse should be aware of which fact about the amniotic fluid? a. It serves as a source of oral fluid and a repository for waste from the fetus. b. The volume remains about the same throughout the term of a healthy pregnancy. c. A volume of less than 300 mL is associated with gastrointestinal malformations. d. A volume of more than 2 L is associated with fetal renal abnormalities.

ANS: A Amniotic fluid serves as a source of oral fluid, serves as a repository for waste from the fetus, cushions the fetus, and helps maintain a constant body temperature. The volume of amniotic fluid changes constantly. Too little amniotic fluid (oligohydramnios) is associated with renal abnormalities. Too much amniotic fluid (hydramnios) is associated with gastrointestinal and other abnormalities.

A key finding from the Human Genome Project is: a. approximately 20,500 genes make up the genome. b. all human beings are 80.99% identical at the DNA level. c. human genes produce only one protein per gene; other mammals produce three proteins per gene. d. single gene testing will become a standardized test for all pregnant patients in the future.

ANS: A Approximately 20,500 genes make up the human genome; this is only twice as many as make up the genomes of roundworms and flies. Human beings are 99.9% identical at the DNA level. Most human genes produce at least three proteins. Single gene testing (e.g., alpha-fetoprotein) is already standardized for prenatal care.

Which occurrence is associated with cervical dilation and effacement? a. Bloody show b. False labor c. Lightening d. Bladder distention

ANS: A As the cervix begins to soften, dilate, and efface, expulsion of the mucous plug that sealed the cervix during pregnancy occurs. This causes rupture of small cervical capillaries. Cervical dilation and effacement do not occur with false labor. Lightening is the descent of the fetus toward the pelvic inlet before labor. Bladder distention occurs when the bladder is not emptied frequently. It may slow down the descent of the fetus during labor.

The nurse caring for the laboring woman should know that meconium is produced by: a. fetal intestines. b. fetal kidneys. c. amniotic fluid. d. the placenta.

ANS: A As the fetus nears term, fetal waste products accumulate in the intestines as dark green-to-black, tarry meconium.

During a patient's physical examination the nurse notes that the lower uterine segment is soft on palpation. The nurse would document this finding as: a. Hegar's sign. b. McDonald's sign. c. Chadwick's sign. d. Goodell's sign.

ANS: A At approximately 6 weeks of gestation, softening and compressibility of the lower uterine segment occurs; this is called Hegar's sign. McDonald's sign indicates a fast food restaurant. Chadwick's sign is the blue-violet coloring of the cervix caused by increased vascularity; this occurs around the fourth week of gestation. Softening of the cervical tip is called Goodell's sign, which may be observed around the sixth week of pregnancy.

With regard to medications, herbs, shots, and other substances normally encountered by pregnant women, the maternity nurse should be aware that: a. both prescription and over-the-counter (OTC) drugs that otherwise are harmless can be made hazardous by metabolic deficiencies of the fetus. b. the greatest danger of drug-caused developmental deficits in the fetus is seen in the final trimester. c. killed-virus vaccines (e.g., tetanus) should not be given during pregnancy, but live-virus vaccines (e.g., measles) are permissible. d. no convincing evidence exists that secondhand smoke is potentially dangerous to the fetus.

ANS: A Both prescription and OTC drugs that otherwise are harmless can be made hazardous by metabolic deficiencies of the fetus. This is especially true for new medications and combinations of drugs. The greatest danger of drug-caused developmental defects exists in the interval from fertilization through the first trimester, when a woman may not realize that she is pregnant. Live-virus vaccines should be part of after birth care; killed-virus vaccines may be administered during pregnancy. Secondhand smoke is associated with fetal growth restriction and increases in infant mortality.

Which statement regarding acronyms in nutrition is accurate? a. Dietary reference intakes (DRIs) consist of recommended dietary allowances (RDAs), adequate intakes (AIs), and upper limits (ULs). b. RDAs are the same as ULs except with better data. c. AIs offer guidelines for avoiding excessive amounts of nutrients. d. They all refer to green leafy vegetables, whole grains, and fruit.

ANS: A DRIs consist of RDAs, AIs, and ULs. AIs are similar to RDAs except that they deal with nutrients about which data are insufficient for certainty (RDA status). ULs are guidelines for avoiding excesses of nutrients for which excess is toxic. Green leafy vegetables, whole grains, and fruit are important, but they are not the whole nutritional story.

The nurse would expect which maternal cardiovascular finding during labor? a. Increased cardiac output b. Decreased pulse rate c. Decreased white blood cell (WBC) count d. Decreased blood pressure

ANS: A During each contraction, 400 mL of blood is emptied from the uterus into the maternal vascular system. This increases cardiac output by about 51% above baseline pregnancy values at term. The heart rate increases slightly during labor. The WBC count can increase during labor. During the first stage of labor, uterine contractions cause systolic readings to increase by about 10 mm Hg. During the second stage, contractions may cause systolic pressures to increase by 30 mm Hg and diastolic readings to increase by 25 mm Hg.

Which nutritional recommendation about fluids is accurate? a. A woman's daily intake should be 8 to 10 glasses (2.3 L) of water, milk, or juice. b. Coffee should be limited to no more than two cups, but tea and cocoa can be consumed without worry. c. Of the artificial sweeteners, only aspartame has been not associated with any maternity health concerns. d. Water with fluoride is especially encouraged because it reduces the child's risk of tooth decay.

ANS: A Eight to ten glasses is the standard for fluids; however, they should be the right fluids. All beverages containing caffeine, including tea, cocoa, and some soft drinks should be avoided or drunk only in limited amounts. Artificial sweeteners, including aspartame, have no ill effects on the normal mother or fetus; however, mothers with phenylketonuria should avoid aspartame. No evidence indicates that prenatal fluoride consumption reduces childhood tooth decay.

A woman is in her seventh month of pregnancy. She has been reporting nasal congestion and occasional epistaxis. The nurse suspects that: a. this is a normal respiratory change in pregnancy caused by elevated levels of estrogen. b. this is an abnormal cardiovascular change, and the nosebleeds are an ominous sign. c. the woman is a victim of domestic violence and is being hit in the face by her partner. d. the woman has been using cocaine intranasally.

ANS: A Elevated levels of estrogen cause capillaries to become engorged in the respiratory tract. This may result in edema in the nose, larynx, trachea, and bronchi. This congestion may cause nasal stuffiness and epistaxis. Cardiovascular changes in pregnancy may cause edema in lower extremities. Determining that the woman is a victim of domestic violence and was hit in the face cannot be made on the basis of the sparse facts provided. If the woman had been hit in the face, she most likely would have additional physical findings. Determination of the use of cocaine by the woman cannot be made on the basis of the sparse facts provided.

The placenta allows exchange of oxygen, nutrients, and waste products between the mother and fetus by: a. contact between maternal blood and fetal capillaries within the chorionic villi. b. interaction of maternal and fetal pH levels within the endometrial vessels. c. a mixture of maternal and fetal blood within the intervillous spaces. d. passive diffusion of maternal carbon dioxide and oxygen into the fetal capillaries.

ANS: A Fetal capillaries within the chorionic villi are bathed with oxygen-rich and nutrient-rich maternal blood within the intervillous spaces. The endometrial vessels are part of the uterus. There is no interaction with the fetal blood at this point. Maternal and fetal bloods do not normally mix. Maternal carbon dioxide does not enter into the fetal circulation.

Which description of the four stages of labor is correct for both definition and duration? a. First stage: onset of regular uterine contractions to full dilation; less than 1 hour to 20 hours b. Second stage: full effacement to 4 to 5 cm; visible presenting part; 1 to 2 hours c. Third state: active pushing to birth; 20 minutes (multiparous women), 50 minutes (first-timer) d. Fourth stage: delivery of the placenta to recovery; 30 minutes to 1 hour

ANS: A Full dilation may occur in less than 1 hour, but in first-time pregnancy it can take up to 20 hours. The second stage extends from full dilation to birth and takes an average of 20 to 50 minutes, although 2 hours is still considered normal. The third stage extends from birth to expulsion of the placenta and usually takes a few minutes. The fourth stage begins after expulsion of the placenta and lasts until homeostasis is reestablished (about 2 hours).

With regard to protein in the diet of pregnant women, nurses should be aware that: a. many protein-rich foods are also good sources of calcium, iron, and B vitamins. b. many women need to increase their protein intake during pregnancy. c. as with carbohydrates and fat, no specific recommendations exist for the amount of protein in the diet. d. high-protein supplements can be used without risk by women on macrobiotic diets.

ANS: A Good protein sources such as meat, milk, eggs, and cheese have a lot of calcium and iron. Most women already eat a high-protein diet and do not need to increase their intake. Protein is sufficiently important that specific servings of meat and dairy are recommended. High-protein supplements are not recommended because they have been associated with an increased incidence of preterm births.

A pregnant woman reports that she is still playing tennis at 32 weeks of gestation. The nurse would be most concerned that during and after tennis matches this woman consumes: a. several glasses of fluid. b. extra protein sources such as peanut butter. c. salty foods to replace lost sodium. d. easily digested sources of carbohydrate.

ANS: A If no medical or obstetric problems contraindicate physical activity, pregnant women should get 30 minutes of moderate physical exercise daily. Liberal amounts of fluid should be consumed before, during, and after exercise because dehydration can trigger premature labor. The woman's calorie intake should be sufficient to meet the increased needs of pregnancy and the demands of exercise.

Many parents-to-be have questions about multiple births. Maternity nurses should be able to tell them that: a. twinning and other multiple births are increasing because of the use of fertility drugs and delayed childbearing. b. dizygotic twins (two fertilized ova) have the potential to be conjoined twins. c. identical twins are more common in white families. d. fraternal twins are same gender, usually male.

ANS: A If the parents-to-be are older and have taken fertility drugs, they would be very interested to know about twinning and other multiple births. Conjoined twins are monozygotic; they are from a single fertilized ovum in which division occurred very late. Identical twins show no racial or ethnic preference; fraternal twins are more common among African-American women. Fraternal twins can be different genders or the same gender. Identical twins are the same gender.

Which presentation is described accurately in terms of both presenting part and frequency of occurrence? a. Cephalic: occiput; at least 95% b. Breech: sacrum; 10% to 15% c. Shoulder: scapula; 10% to 15% d. Cephalic: cranial; 80% to 85%

ANS: A In cephalic presentations (head first), the presenting part is the occiput; this occurs in 96% of births. In a breech birth, the sacrum emerges first; this occurs in about 3% of births. In shoulder presentations, the scapula emerges first; this occurs in only 1% of births.

The nurse is assessing the knowledge of new parents with a child born with maple syrup urine disease (MSUD). This is an autosomal recessive inherited disorder, which means that: a. both genes of a pair must be abnormal for the disorder to be expressed. b. only one copy of the abnormal gene is required for the disorder to be expressed. c. the disorder occurs in males and heterozygous females. d. the disorder is carried on the X chromosome.

ANS: A MSUD is a type of autosomal recessive inheritance disorder in which both genes of a pair must be abnormal for the disorder to be expressed. MSUD is not an X-linked dominant or recessive disorder or an autosomal dominant inheritance disorder.

With regard to prenatal genetic testing, nurses should be aware that: a. maternal serum screening can determine whether a pregnant woman is at risk of carrying a fetus with Down syndrome. b. carrier screening tests look for gene mutations of people already showing symptoms of a disease. c. predisposition testing predicts with near certainty that symptoms will appear. d. presymptomatic testing is used to predict the likelihood of breast cancer.

ANS: A Maternal serum screening identifies the risk for the neural tube defect and the specific chromosome abnormality involved in Down syndrome. Carriers of some diseases, such as sickle cell disease, do not display symptoms. Predisposition testing determines susceptibility, such as for breast cancer. presymptomatic testing indicates that symptoms are certain to appear if the gene is present.

The nurse caring for a pregnant patient knows that her health teaching regarding fetal circulation has been effective when the patient reports that she has been sleeping: a. in a side-lying position. b. on her back with a pillow under her knees. c. with the head of the bed elevated. d. on her abdomen.

ANS: A Optimal circulation is achieved when the woman is lying at rest on her side. Decreased uterine circulation may lead to intrauterine growth restriction. Previously it was believed that the left lateral position promoted maternal cardiac output, enhancing blood flow to the fetus. However, it is now known that the side-lying position enhances uteroplacental blood flow. If a woman lies on her back with the pressure of the uterus compressing the vena cava, blood return to the right atrium is diminished. Although having the head of the bed elevated is recommended and ideal for later in pregnancy, the woman still must maintain a lateral tilt to the pelvis to avoid compression of the vena cava. Many women find lying on her abdomen uncomfortable as pregnancy advances. Side-lying is the ideal position to promote blood flow to the fetus.

A new mother asks the nurse about the "white substance" covering her infant. The nurse explains that the purpose of vernix caseosa is to: a. protect the fetal skin from amniotic fluid. b. promote normal peripheral nervous system development. c. allow transport of oxygen and nutrients across the amnion. d. regulate fetal temperature.

ANS: A Prolonged exposure to amniotic fluid during the fetal period could result in breakdown of the skin without the protection of the vernix caseosa. Normal development of the peripheral nervous system is dependent on nutritional intake of the mother. The amnion is the inner membrane that surrounds the fetus. It is not involved in the oxygen and nutrient exchange. The amniotic fluid aids in maintaining fetal temperature.

After you complete your nutritional counseling for a pregnant woman, you ask her to repeat your instructions so you can assess her understanding of the instructions given. Which statement indicates that she understands the role of protein in her pregnancy? a. "Protein will help my baby grow." b. "Eating protein will prevent me from becoming anemic." c. "Eating protein will make my baby have strong teeth after he/she is born." d. "Eating protein will prevent me from being diabetic."

ANS: A Protein is the nutritional element basic to growth. An adequate protein intake is essential to meeting the increasing demands of pregnancy. These demands arise from the rapid growth of the fetus; the enlargement of the uterus, mammary glands, and placenta; the increase in the maternal blood volume; and the formation of amniotic fluid. Iron intake prevents anemia. Calcium intake is needed for fetal bone and tooth development. Glycemic control is needed in diabetics; protein is one nutritional factor to consider, but this is not the primary role of protein intake.

To help a woman reduce the severity of nausea caused by morning sickness, the nurse might suggest that she: a. try a tart food or drink such as lemonade or salty foods such as potato chips. b. drink plenty of fluids early in the day. c. brush her teeth immediately after eating. d. never snack before bedtime.

ANS: A Some women can tolerate tart or salty foods when they are nauseous. The woman should avoid drinking too much when nausea is most likely, but she should make up the fluid levels later in the day when she feels better. The woman should avoid brushing her teeth immediately after eating. A small snack of cereal and milk or yogurt before bedtime may help the stomach in the morning.

Prenatal testing for human immunodeficiency virus (HIV) is recommended for: a. all women, regardless of risk factors. b. a woman who has had more than one sexual partner. c. a woman who has had a sexually transmitted infection. d. a woman who is monogamous with her partner.

ANS: A Testing for the antibody to HIV is strongly recommended for all pregnant women. A HIV test is recommended for all women, regardless of risk factors. Women who test positive for HIV can be treated, reducing the risk of transmission to the fetus.

Appendicitis may be difficult to diagnose in pregnancy because the appendix is: a. displaced upward and laterally, high and to the right. b. displaced upward and laterally, high and to the left. c. deep at McBurney point. d. displaced downward and laterally, low and to the right.

ANS: A The appendix is displaced high and to the right, beyond McBurney point.

To care for a laboring woman adequately, the nurse understands that the __________ stage of labor varies the most in length? a. first b. second c. third d. fourth

ANS: A The first stage of labor is considered to last from the onset of regular uterine contractions to full dilation of the cervix. The first stage is much longer than the second and third stages combined. In a first-time pregnancy the first stage of labor can take up to 20 hours. The second stage of labor lasts from the time the cervix is fully dilated to the birth of the fetus. The average length is 20 minutes for a multiparous woman and 50 minutes for a nulliparous woman. The third stage of labor lasts from the birth of the fetus until the placenta is delivered. This stage may be as short as 3 minutes or as long as 1 hour. The fourth stage of labor, recovery, lasts about 2 hours after delivery of the placenta.

The nurse must be cognizant that an individual's genetic makeup is known as his or her: a. genotype. b. phenotype. c. karyotype. d. chromotype.

ANS: A The genotype comprises all the genes the individual can pass on to a future generation. The phenotype is the observable expression of an individual's genotype. The karyotype is a pictorial analysis of the number, form, and size of an individual's chromosomes. Genotype refers to an individual's genetic makeup.

Which behavior indicates that a woman is "seeking safe passage" for herself and her infant? a. She keeps all prenatal appointments. b. She "eats for two." c. She drives her car slowly. d. She wears only low-heeled shoes.

ANS: A The goal of prenatal care is to foster a safe birth for the infant and mother. Although eating properly, driving carefully, and using proper body mechanics all are healthy measures that a mother can take, obtaining prenatal care is the optimal method for providing safety for both herself and her baby.

While teaching the expectant mother about personal hygiene during pregnancy, maternity nurses should be aware that: a. tub bathing is permitted even in late pregnancy unless membranes have ruptured. b. the perineum should be wiped from back to front. c. bubble bath and bath oils are permissible because they add an extra soothing and cleansing action to the bath. d. expectant mothers should use specially treated soap to cleanse the nipples.

ANS: A The main danger from taking baths is falling in the tub. The perineum should be wiped from front to back. Bubble baths and bath oils should be avoided because they may irritate the urethra. Soap, alcohol, ointments, and tinctures should not be used to cleanse the nipples because they remove protective oils. Warm water is sufficient.

The multiple marker test is used to assess the fetus for which condition? a. Down syndrome b. Diaphragmatic hernia c. Congenital cardiac abnormality d. Anencephaly

ANS: A The maternal serum level of alpha-fetoprotein is used to screen for Down syndrome, neural tube defects, and other chromosome anomalies. The multiple marker test would not detect diaphragmatic hernia, congenital cardiac abnormality, or anencephaly. Additional testing, such as ultrasonography and amniocentesis, would be required to diagnose these conditions.

The mucous plug that forms in the endocervical canal is called the: a. operculum. b. leukorrhea. c. funic souffle. d. ballottement.

ANS: A The operculum protects against bacterial invasion. Leukorrhea is the mucus that forms the endocervical plug (the operculum). The funic souffle is the sound of blood flowing through the umbilical vessels. Ballottement is a technique for palpating the fetus.

In relation to primary and secondary powers, the maternity nurse comprehends that: a. primary powers are responsible for effacement and dilation of the cervix. b. effacement generally is well ahead of dilation in women giving birth for the first time; they are closer together in subsequent pregnancies. c. scarring of the cervix caused by a previous infection or surgery may make the delivery a bit more painful, but it should not slow or inhibit dilation. d. pushing in the second stage of labor is more effective if the woman can breathe deeply and control some of her involuntary needs to push, as the nurse directs.

ANS: A The primary powers are responsible for dilation and effacement; secondary powers are concerned with expulsion of the fetus. Effacement generally is well ahead of dilation in first-timers; they are closer together in subsequent pregnancies. Scarring of the cervix may slow dilation. Pushing is more effective and less fatiguing when the woman begins to push only after she has the urge to do so.

A first-time mother at 18 weeks of gestation comes for her regularly scheduled prenatal visit. The patient tells the nurse that she is afraid that she is going into premature labor because she is beginning to have regular contractions. The nurse explains that this is the Braxton Hicks sign and teaches the patient that this type of contraction: a. is painless. b. increases with walking. c. causes cervical dilation. d. impedes oxygen flow to the fetus.

ANS: A Uterine contractions can be felt through the abdominal wall soon after the fourth month of gestation. Braxton Hicks contractions are regular and painless and continue throughout the pregnancy. Although they are not painful, some women complain that they are annoying. Braxton Hicks contractions usually cease with walking or exercise. They can be mistaken for true labor; however, they do not increase in intensity or frequency or cause cervical dilation. In addition, they facilitate uterine blood flow through the intervillous spaces of the placenta and promote oxygen delivery to the fetus.

Signs and symptoms that a woman should report immediately to her health care provider include: (Select all that apply.) a. vaginal bleeding. b. rupture of membranes. c. heartburn accompanied by severe headache. d. decreased libido. e. Urinary frequency.

ANS: A, B, C Vaginal bleeding, rupture of membranes, and severe headaches all are signs of potential complications in pregnancy. Patients should be advised to report these signs to the health care provider. Decreased libido and urinary frequency are common discomforts of pregnancy that do not require immediate health care interventions.

Most women with uncomplicated pregnancies can use the nurse as their primary source for nutritional information. The nurse or midwife should refer a patient to a registered dietitian for in-depth nutritional counseling in the following situations: (Select all that apply.) a. preexisting or gestational illness such as diabetes. b. ethnic or cultural food patterns. c. obesity. d. vegetarian diet. e. allergy to tree nuts.

ANS: A, B, C, D The nurse should be especially aware that conditions such as diabetes can require in-depth dietary planning and evaluation. To prevent issues with hypoglycemia and hyperglycemia and an increased risk for perinatal morbidity and mortality, this patient would benefit from a referral to a dietitian. Consultation with a dietitian may ensure that cultural food beliefs are congruent with modern knowledge of fetal development and that adjustments can be made to ensure that all nutritional needs are met. The obese pregnant patient may be under the misapprehension that because of her excess weight little or no weight gain is necessary. According to the Institute of Medicine, a patient with a body mass index in the obese range should gain at least 7 kg to ensure a healthy outcome. This patient may require in-depth counseling on optimal food choices. The vegetarian patient needs to have her dietary intake carefully assessed to ensure that the optimal combination of amino acids and protein intake is achieved. Very strict vegetarians (vegans) who consume only plant products may also require vitamin B and mineral supplementation. A patient with a food allergy would not alter that component of her diet during pregnancy; therefore, no additional consultation is necessary.`

Congenital disorders refer to conditions that are present at birth. These disorders may be inherited and caused by environmental factors or maternal malnutrition. Toxic exposures have the greatest effect on development between 15 and 60 days of gestation. For the nurse to be able to conduct a complete assessment of the newly pregnant patient, she should understand the significance of exposure to known human teratogens. These include: (Select all that apply.) a. infections. b. radiation. c. maternal conditions. d. drugs. e. chemicals.

ANS: A, B, C, D, E Exposure to radiation and numerous infections may result in profound congenital deformities. These include but are not limited to varicella, rubella, syphilis, parvovirus, cytomegalovirus, and toxoplasmosis. Certain maternal conditions such as diabetes and phenylketonuria may also affect organs and other parts of the embryo during this developmental period. Drugs such as antiseizure medication and some antibiotics as well as chemicals, including lead, mercury, tobacco, and alcohol, also may result in structural and functional abnormalities.

Which congenital malformations result from multifactorial inheritance? (Select all that apply.) a. Cleft lip b. Congenital heart disease c. Cri du chat syndrome d. Anencephaly e. Pyloric stenosis

ANS: A, B, D, E All these congenital malformations are associated with multifactorial inheritance. Cri du chat syndrome is related to a chromosome deletion.

Which factors influence cervical dilation? (Select all that apply.) a. Strong uterine contractions. b. The force of the presenting fetal part against the cervix. c. The size of the female pelvis. d. The pressure applied by the amniotic sac. e. Scarring of the cervix.

ANS: A, B, D, E Dilation of the cervix occurs by the drawing upward of the musculofibrous components of the cervix, which is caused by strong uterine contractions. Pressure exerted by the amniotic fluid while the membranes are intact or by the force applied by the presenting part also can promote cervical dilation. Scarring of the cervix as a result of a previous infection or surgery may slow cervical dilation. Pelvic size does not affect cervical dilation.

A woman has just moved to the United States from Mexico. She is 3 months pregnant and has arrived for her first prenatal visit. During her assessment interview, you discover that she has not had any immunizations. Which immunizations should she receive at this point in her pregnancy? (Select all that apply.) a. Tetanus b. Diphtheria c. Chickenpox d. Rubella e. Hepatitis B

ANS: A, B, E Immunization with live or attenuated live viruses is contraindicated during pregnancy because of potential teratogenicity. Vaccines consisting of killed viruses may be used. Immunizations that may be administered during pregnancy include tetanus, diphtheria, recombinant hepatitis B, and rabies vaccines. Live-virus vaccines include those for measles (rubeola and rubella), chickenpox, and mumps.

The diagnosis of pregnancy is based on which positive signs of pregnancy? (Select all that apply.) a. Identification of fetal heartbeat b. Palpation of fetal outline c. Visualization of the fetus d. Verification of fetal movement e. Positive hCG test

ANS: A, C, D Identification of fetal heartbeat, visualization of the fetus, and verification of fetal movement are all positive, objective signs of pregnancy. Palpation of fetal outline and a positive hCG test are probable signs of pregnancy. A tumor also can be palpated. Medication and tumors may lead to false-positive results on pregnancy tests.

Signs that precede labor include: (Select all that apply.) a. lightening. b. exhaustion. c. bloody show. d. rupture of membranes. e. decreased fetal movement.

ANS: A, C, D Signs that precede labor may include lightening, urinary frequency, backache, weight loss, surge of energy, bloody show, and rupture of membranes. Many women experience a burst of energy before labor. A decrease in fetal movement is an ominous sign that does not always correlate with labor.

Along with gas exchange and nutrient transfer, the placenta produces many hormones necessary for normal pregnancy. These include: (Select all that apply.) a. human chorionic gonadotropin (hCG). b. insulin. c. estrogen. d. progesterone. e. testosterone.

ANS: A, C, D hCG causes the corpus luteum to persist and produce the necessary estrogens and progesterone for the first 6 to 8 weeks. Estrogens cause enlargement of the woman's uterus and breasts; cause growth of the ductal system in the breasts; and, as term approaches, play a role in the initiation of labor. Progesterone causes the endometrium to change, providing early nourishment. Progesterone also protects against spontaneous abortion by suppressing maternal reactions to fetal antigens and reduces unnecessary uterine contractions. Other hormones produced by the placenta include hCT, hCA, and numerous growth factors. Human placental lactogen promotes normal nutrition and growth of the fetus and maternal breast development for lactation. This hormone decreases maternal insulin sensitivity and utilization of glucose, making more glucose available for fetal growth. If a Y chromosome is present in the male fetus, hCG causes the fetal testes to secrete testosterone necessary for the normal development of male reproductive structures.

What type of cultural concern is the most likely deterrent to many women seeking prenatal care? a. Religion b. Modesty c. Ignorance d. Belief that physicians are evil

ANS: B A concern for modesty is a deterrent to many women seeking prenatal care. For some women, exposing body parts, especially to a man, is considered a major violation of their modesty. Many cultural variations are found in prenatal care. Even if the prenatal care described is familiar to a woman, some practices may conflict with the beliefs and practices of a subculture group to which she belongs.

With regard to nutritional needs during lactation, a maternity nurse should be aware that: a. the mother's intake of vitamin C, zinc, and protein now can be lower than during pregnancy. b. caffeine consumed by the mother accumulates in the infant, who may be unusually active and wakeful. c. critical iron and folic acid levels must be maintained. d. lactating women can go back to their prepregnant calorie intake.

ANS: B A lactating woman needs to avoid consuming too much caffeine. Vitamin C, zinc, and protein levels need to be moderately higher during lactation than during pregnancy. The recommendations for iron and folic acid are lower during lactation. Lactating women should consume about 500 kcal more than their prepregnancy intake, at least 1800 kcal daily overall

Which statement made by a lactating woman would lead the nurse to believe that the woman might have lactose intolerance? a. "I always have heartburn after I drink milk." b. "If I drink more than a cup of milk, I usually have abdominal cramps and bloating." c. "Drinking milk usually makes me break out in hives." d. "Sometimes I notice that I have bad breath after I drink a cup of milk."

ANS: B Abdominal cramps and bloating are consistent with lactose intolerance. One problem that can interfere with milk consumption is lactose intolerance, which is the inability to digest milk sugar because of a lack of the enzyme lactase in the small intestine. Milk consumption may cause abdominal cramping, bloating, and diarrhea in people who are lactose intolerant, although many affected individuals can tolerate small amounts of milk without symptoms

A woman who is 32 weeks' pregnant is informed by the nurse that a danger sign of pregnancy could be: a. constipation. b. alteration in the pattern of fetal movement. c. heart palpitations. d. edema in the ankles and feet at the end of the day.

ANS: B An alteration in the pattern or amount of fetal movement may indicate fetal jeopardy. Constipation, heart palpitations, and ankle and foot edema are normal discomforts of pregnancy that occur in the second and third trimesters.

The musculoskeletal system adapts to the changes that occur during pregnancy. A woman can expect to experience what change? a. Her center of gravity will shift backward. b. She will have increased lordosis. c. She will have increased abdominal muscle tone. d. She will notice decreased mobility of her pelvic joints.

ANS: B An increase in the normal lumbosacral curve (lordosis) develops, and a compensatory curvature in the cervicodorsal region develops to help the woman maintain her balance. The center of gravity shifts forward. She will have decreased muscle tone. She will notice increased mobility of her pelvic joints.

With regard to chromosome abnormalities, nurses should be aware that: a. they occur in approximately 10% of newborns. b. abnormalities of number are the leading cause of pregnancy loss. c. Down syndrome is a result of an abnormal chromosome structure. d. unbalanced translocation results in a mild abnormality that the child will outgrow.

ANS: B Aneuploidy is an abnormality of number that also is the leading genetic cause of mental retardation. Chromosome abnormalities occur in less than 1% of newborns. Down syndrome is the most common form of trisomal abnormality, an abnormality of chromosome number (47 chromosomes). Unbalanced translocation is an abnormality of chromosome structure that often has serious clinical effects.

A 31-year-old woman believes that she may be pregnant. She took an OTC pregnancy test 1 week ago after missing her period; the test was positive. During her assessment interview, the nurse enquires about the woman's last menstrual period and asks whether she is taking any medications. The woman states that she takes medicine for epilepsy. She has been under considerable stress lately at work and has not been sleeping well. She also has a history of irregular periods. Her physical examination does not indicate that she is pregnant. She has an ultrasound scan that reveals she is not pregnant. What is the most likely cause of the false-positive pregnancy test result? a. She took the pregnancy test too early. b. She takes anticonvulsants. c. She has a fibroid tumor. d. She has been under considerable stress and has a hormone imbalance.

ANS: B Anticonvulsants may cause false-positive pregnancy test results. OTC pregnancy tests use enzyme-linked immunosorbent assay technology, which can yield positive results 4 days after implantation. Implantation occurs 6 to 10 days after conception. If the woman were pregnant, she would be into her third week at this point (having missed her period 1 week ago). Fibroid tumors do not produce hormones and have no bearing on hCG pregnancy tests. Although stress may interrupt normal hormone cycles (menstrual cycles), it does not affect human chorionic gonadotropin levels or produce positive pregnancy test results.

Three servings of milk, yogurt, or cheese plus two servings of meat, poultry, or fish adequately supply the recommended amount of protein for a pregnant woman. Many patients are concerned about the increased levels of mercury in fish and may be afraid to include this source of nutrients in their diet. Sound advice by the nurse to assist the patient in determining which fish is safe to consume would include: a. canned white tuna is a preferred choice. b. avoid shark, swordfish, and mackerel. c. fish caught in local waterways are the safest. d. salmon and shrimp contain high levels of mercury.

ANS: B As a precaution, the pregnant patient should avoid eating all of these and the less common tilefish. High levels of mercury can harm the developing nervous system of the fetus. It is essential for the nurse to assist the patient in understanding the differences between numerous sources of this product. A pregnant patient can take 12 ounces a week of canned light tuna; however, canned white, albacore, or tuna steaks contain higher levels of mercury and should be limited to no more than 6 ounces per week. It is a common misconception that fish caught in local waterways are the safest. Pregnant women and mothers of young children should check with local advisories about the safety of fish caught by families and friends in nearby bodies of water. If no information is available, these fish sources should be avoided, limited to less than 6 ounces, or the only fish consumed that week. Commercially caught fish that are low in mercury include salmon, shrimp, pollock, or catfish.

To reassure and educate pregnant patients about changes in their cardiovascular system, maternity nurses should be aware that: a. a pregnant woman experiencing disturbed cardiac rhythm, such as sinus arrhythmia requires close medical and obstetric observation, no matter how healthy she otherwise may appear. b. changes in heart size and position and increases in blood volume create auditory changes from 20 weeks to term. c. palpitations are twice as likely to occur in twin gestations. d. all of the above changes will likely occur.

ANS: B Auscultatory changes should be discernible after 20 weeks of gestation. A healthy woman with no underlying heart disease does not need any therapy. The maternal heart rate increases in the third trimester, but palpitations may not occur. Auditory changes are discernible at 20 weeks.

Cardiovascular system changes occur during pregnancy. Which finding would be considered normal for a woman in her second trimester? a. Less audible heart sounds (S1, S2) b. Increased pulse rate c. Increased blood pressure d. Decreased red blood cell (RBC) production

ANS: B Between 14 and 20 weeks of gestation, the pulse increases about 10 to 15 beats/min, which persists to term. Splitting of S1 and S2 is more audible. In the first trimester, blood pressure usually remains the same as at the prepregnancy level, but it gradually decreases up to about 20 weeks of gestation. During the second trimester, both the systolic and the diastolic pressures decrease by about 5 to 10 mm Hg. Production of RBCs accelerates during pregnancy.

Women with an inadequate weight gain during pregnancy are at higher risk of giving birth to an infant with: a. spina bifida. b. intrauterine growth restriction. c. diabetes mellitus. d. Down syndrome.

ANS: B Both normal-weight and underweight women with inadequate weight gain have an increased risk of giving birth to an infant with intrauterine growth restriction. Spina bifida, diabetes mellitus, and Down syndrome are not associated with inadequate maternal weight gain.

A 3-year-old girl's mother is 6 months pregnant. What concern is this child likely to verbalize? a. How the baby will "get out"? b. What the baby will eat? c. Whether her mother will die? d. What color eyes the baby has?

ANS: B By age 3 or 4, children like to be told the story of their own beginning and accept its comparison with the present pregnancy. They like to listen to the fetal heartbeat and feel the baby move. Sometimes they worry about how the baby is being fed and what it wears. School-age children take a more clinical interest in their mother's pregnancy and may want to know, "How did the baby get in there?" and "How will it get out?" Whether her mother will die does not tend to be the focus of a child's questions about the impending birth of a sibling. The baby's eye color does not tend to be the focus of children's questions about the impending birth of a sibling.

In presenting to obstetric nurses interested in genetics, the genetic nurse identifies the primary risk(s) associated with genetic testing as: a. anxiety and altered family relationships. b. denial of insurance benefits. c. high false-positive results associated with genetic testing. d. ethnic and socioeconomic disparity associated with genetic testing.

ANS: B Decisions about genetic testing are shaped by socioeconomic status and the ability to pay for the testing. Some types of genetic testing are expensive and are not covered by insurance benefits. Anxiety and altered family relationships, high false-positive results, and ethnic and socioeconomic disparity are factors that may be difficulties associated with genetic testing, but they are not risks associated with testing.

Which statement is the best rationale for assessing maternal vital signs between contractions? a. During a contraction, assessing fetal heart rates is the priority. b. Maternal circulating blood volume increases temporarily during contractions. c. Maternal blood flow to the heart is reduced during contractions. d. Vital signs taken during contractions are not accurate.

ANS: B During uterine contractions, blood flow to the placenta temporarily stops, causing a relative increase in the mother's blood volume, which in turn temporarily increases blood pressure and slows pulse. It is important to monitor fetal response to contractions; however, this question is concerned with the maternal vital signs. Maternal blood flow is increased during a contraction. Vital signs are altered by contractions but are considered accurate for that period of time.

A pregnant woman experiencing nausea and vomiting should: a. drink a glass of water with a fat-free carbohydrate before getting out of bed in the morning. b. eat small, frequent meals (every 2 to 3 hours). c. increase her intake of high-fat foods to keep the stomach full and coated. d. limit fluid intake throughout the day.

ANS: B Eating small, frequent meals is the correct suggestion for a woman experiencing nausea and vomiting. A pregnant woman experiencing nausea and vomiting should avoid consuming fluids early in the day or when nauseated, but should compensate by drinking fluids at other times. A pregnant woman experiencing nausea and vomiting should reduce her intake of fried and other fatty foods.

The slight overlapping of cranial bones or shaping of the fetal head during labor is called: a. lightening. b. molding. c. Ferguson reflex. d. Valsalva maneuver.

ANS: B Fetal head formation is called molding. Molding also permits adaptation to various diameters of the maternal pelvis. Lightening is the mother's sensation of decreased abdominal distention, which usually occurs the week before labor. The Ferguson reflex is the contraction urge of the uterus after stimulation of the cervix. The Valsalva maneuver describes conscious pushing during the second stage of labor.

To reassure and educate pregnant patients about the functioning of their kidneys in eliminating waste products, maternity nurses should be aware that: a. increased urinary output makes pregnant women less susceptible to urinary infection. b. increased bladder sensitivity and then compression of the bladder by the enlarging uterus results in the urge to urinate even if the bladder is almost empty. c. renal (kidney) function is more efficient when the woman assumes a supine position. d. using diuretics during pregnancy can help keep kidney function regular.

ANS: B First bladder sensitivity and then compression of the bladder by the uterus result in the urge to urinate more often. Numerous anatomic changes make a pregnant woman more susceptible to urinary tract infection. Renal function is more efficient when the woman lies in the lateral recumbent position and less efficient when she is supine. Diuretic use during pregnancy can overstress the system and cause problems.

Which time-based description of a stage of development in pregnancy is accurate? a. Viability—22 to 37 weeks since the last menstrual period (LMP) (assuming a fetal weight >500 g). b. Full Term—Pregnancy from the beginning of week 39 of gestation to the end of week 40. c. Preterm—Pregnancy from 20 to 28 weeks. d. Postdate—Pregnancy that extends beyond 38 weeks.

ANS: B Full Term is 39 to 40 weeks of gestation. Viability is the ability of the fetus to live outside the uterus before coming to term, or 22 to 24 weeks since LMP. Preterm is 20 to 37 weeks of gestation. Postdate or postterm is a pregnancy that extends beyond 42 weeks or what is considered the limit of full term.

A woman is at 14 weeks of gestation. The nurse would expect to palpate the fundus at which level? a. Not palpable above the symphysis at this time b. Slightly above the symphysis pubis c. At the level of the umbilicus d. Slightly above the umbilicus

ANS: B In normal pregnancies, the uterus grows at a predictable rate. It may be palpated above the symphysis pubis sometime between the 12th and 14th weeks of pregnancy. As the uterus grows, it may be palpated above the symphysis pubis sometime between the 12th and 14th weeks of pregnancy. The uterus rises gradually to the level of the umbilicus at 22 to 24 weeks of gestation.

The most important reason for evaluating the pattern of weight gain in pregnancy is to: a. prevent excessive adipose tissue deposits. b. identify potential nutritional problems or complications of pregnancy. c. assess the need to limit caloric intake in obese women. d. determine cultural influences on the woman's diet.

ANS: B Maternal and fetal risks in pregnancy are increased when the mother is significantly overweight. Excessive adipose tissue may occur with excess weight gain; however, this is not the reason for monitoring the weight gain pattern. It is important to monitor the pattern of weight gain to identify complications. The pattern of weight gain is not influenced by cultural influences.

A woman is 15 weeks pregnant with her first baby. She asks how long it will be before she feels the baby move. The best answer is: a. "You should have felt the baby move by now." b. "Within the next month, you should start to feel fluttering sensations." c. "The baby is moving; however, you can't feel it yet." d. "Some babies are quiet, and you don't feel them move."

ANS: B Maternal perception of fetal movement usually begins 16 to 20 weeks after conception. Because this is her first pregnancy, movement is felt toward the later part of the 16- to 20-week time period. Stating that "you should have felt the baby move by now" is incorrect and may be alarming to the patient. Fetal movement should be felt by 16 to 20 weeks. If movement is not felt by the end of that time, further assessment will be necessary.

During the first trimester, a woman can expect which of the following changes in her sexual desire? a. An increase, because of enlarging breasts b. A decrease, because of nausea and fatigue c. No change d. An increase, because of increased levels of female hormones

ANS: B Maternal physiologic changes such as breast enlargement, nausea, fatigue, abdominal changes, perineal enlargement, leukorrhea, pelvic vasocongestion, and orgasmic responses may affect sexuality and sexual expression. Libido may be depressed in the first trimester but often increases during the second and third trimesters. During pregnancy, the breasts may become enlarged and tender; this tends to interfere with coitus, decreasing the desire to engage in sexual activity.

A patient in her first trimester complains of nausea and vomiting. She asks, "Why does this happen?" The nurse's best response is: a. "It is due to an increase in gastric motility." b. "It may be due to changes in hormones." c. "It is related to an increase in glucose levels." d. "It is caused by a decrease in gastric secretions."

ANS: B Nausea and vomiting are believed to be caused by increased levels of hormones, decreased gastric motility, and hypoglycemia. Gastric motility decreases during pregnancy. Glucose levels decrease in the first trimester. Although gastric secretions decrease, this is not the main cause of nausea and vomiting.

Maternal nutritional status is an especially significant factor of the many factors that influence the outcome of pregnancy because: a. it is very difficult to adjust because of people's ingrained eating habits. b. it is an important preventive measure for a variety of problems. c. women love obsessing about their weight and diets. d. a woman's preconception weight becomes irrelevant.

ANS: B Nutritional status draws so much attention not only for its effect on a healthy pregnancy and birth but also because significant changes are within relatively easy reach.

To prevent gastrointestinal upset, patients should be instructed to take iron supplements: a. on a full stomach. b. at bedtime. c. after eating a meal. d. with milk.

ANS: B Patients should be instructed to take iron supplements at bedtime. Iron supplements are best absorbed if they are taken when the stomach is empty. Bran, tea, coffee, milk, and eggs may reduce absorption. Iron can be taken at bedtime if abdominal discomfort occurs when it is taken between meals.

A pregnant woman at 18 weeks of gestation calls the clinic to report that she has been experiencing occasional backaches of mild-to-moderate intensity. The nurse would recommend that she: a. do Kegel exercises. b. do pelvic rock exercises. c. use a softer mattress. d. stay in bed for 24 hours.

ANS: B Pelvic rock exercises may help stretch and strengthen the abdominal and lower back muscles and relieve low back pain. Kegel exercises increase the tone of the pelvic area, not the back. A softer mattress may not provide the support needed to maintain proper alignment of the spine and may contribute to back pain. Stretching and other exercises to relieve back pain should be performed several times a day.

The nurse teaches a pregnant woman about the presumptive, probable, and positive signs of pregnancy. The woman demonstrates understanding of the nurse's instructions if she states that a positive sign of pregnancy is: a. a positive pregnancy test. b. fetal movement palpated by the nurse-midwife. c. Braxton Hicks contractions. d. quickening.

ANS: B Positive signs of pregnancy are attributed to the presence of a fetus, such as hearing the fetal heartbeat or palpating fetal movement. A positive pregnancy test and Braxton Hicks contractions are probable signs of pregnancy. Quickening is a presumptive sign of pregnancy.

The nurse caring for a newly pregnant woman would advise her that ideally prenatal care should begin: a. before the first missed menstrual period. b. after the first missed menstrual period. c. after the second missed menstrual period. d. after the third missed menstrual period.

ANS: B Prenatal care ideally should begin soon after the first missed menstrual period. Regular prenatal visits offer opportunities to ensure the health of the expectant mother and her infant.

Which finding in the urine analysis of a pregnant woman is considered a variation of normal? a. Proteinuria b. Glycosuria c. Bacteria in the urine d. Ketones in the urine

ANS: B Small amounts of glucose may indicate "physiologic spilling." The presence of protein could indicate kidney disease or preeclampsia. Urinary tract infections are associated with bacteria in the urine. An increase in ketones indicates that the patient is exercising too strenuously or has an inadequate fluid and food intake.

To teach patients about the process of labor adequately, the nurse knows that which event is the best indicator of true labor? a. Bloody show b. Cervical dilation and effacement c. Fetal descent into the pelvic inlet d. Uterine contractions every 7 minutes

ANS: B The conclusive distinction between true and false labor is that contractions of true labor cause progressive change in the cervix. Bloody show can occur before true labor. Fetal descent can occur before true labor. False labor may have contractions that occur this frequently; however, this is usually inconsistent.

A woman's obstetric history indicates that she is pregnant for the fourth time and all of her children from previous pregnancies are living. One was born at 39 weeks of gestation, twins were born at 34 weeks of gestation, and another child was born at 35 weeks of gestation. What is her gravidity and parity using the GTPAL system? a. 3-1-1-1-3 b. 4-1-2-0-4 c. 3-0-3-0-3 d. 4-2-1-0-3

ANS: B The correct calculation of this woman's gravidity and parity is 4-1-2-0-4. The numbers reflect the woman's gravidity and parity information. Using the GPTAL system, her information is calculated as: G: The first number reflects the total number of times the woman has been pregnant; she is pregnant for the fourth time. T: This number indicates the number of pregnancies carried to term, not the number of deliveries at term; only one of her pregnancies has resulted in a fetus at term. P: This is the number of pregnancies that resulted in a preterm birth; the woman has had two pregnancies in which she delivered preterm. A: This number signifies whether the woman has had any abortions or miscarriages before the period of viability; she has not. L: This number signifies the number of children born who are currently living; the woman has four children.

The nurse has received report regarding her patient in labor. The woman's last vaginal examination was recorded as 3 cm, 30%, and -2. The nurse's interpretation of this assessment is that: a. the cervix is effaced 3 cm, it is dilated 30%, and the presenting part is 2 cm above the ischial spines. b. the cervix is 3 cm dilated, it is effaced 30%, and the presenting part is 2 cm above the ischial spines. c. the cervix is effaced 3 cm, it is dilated 30%, and the presenting part is 2 cm below the ischial spines. d. the cervix is dilated 3 cm, it is effaced 30%, and the presenting part is 2 cm below the ischial spines.

ANS: B The correct description of the vaginal examination for this woman in labor is the cervix is 3 cm dilated, it is effaced 30%, and the presenting part is 2 cm above the ischial spines. The sterile vaginal examination is recorded as centimeters of cervical dilation, percentage of cervical dilation, and the relationship of the presenting part to the ischial spines (either above or below).

A man's wife is pregnant for the third time. One child was born with cystic fibrosis, and the other child is healthy. The man wonders what the chance is that this child will have cystic fibrosis. This type of testing is known as: a. occurrence risk. b. recurrence risk. c. predictive testing. d. predisposition testing.

ANS: B The couple already has a child with a genetic disease so they will be given a recurrence risk test. If a couple has not yet had children but are known to be at risk for having children with a genetic disease, they are given an occurrence risk test. Predictive testing is used to clarify the genetic status of an asymptomatic family member. Predisposition testing differs from presymptomatic testing in that a positive result does not indicate 100% risk of a condition developing.

As relates to the structure and function of the placenta, the maternity nurse should be aware that: a. as the placenta widens, it gradually thins to allow easier passage of air and nutrients. b. as one of its early functions, the placenta acts as an endocrine gland. c. the placenta is able to keep out most potentially toxic substances such as cigarette smoke to which the mother is exposed. d. optimal blood circulation is achieved through the placenta when the woman is lying on her back or standing.

ANS: B The placenta produces four hormones necessary to maintain the pregnancy. The placenta widens until week 20 and continues to grow thicker. Toxic substances such as nicotine and carbon monoxide readily cross the placenta into the fetus. Optimal circulation occurs when the woman is lying on her side.

The maternity nurse understands that vascular volume increases 40% to 45% during pregnancy to: a. compensate for decreased renal plasma flow. b. provide adequate perfusion of the placenta. c. eliminate metabolic wastes of the mother. d. prevent maternal and fetal dehydration.

ANS: B The primary function of increased vascular volume is to transport oxygen and nutrients to the fetus via the placenta. Renal plasma flow increases during pregnancy. Assisting with pulling metabolic wastes from the fetus for maternal excretion is one purpose of the increased vascular volume.

To reassure and educate pregnant patients about changes in the uterus, nurses should be aware that: a. lightening occurs near the end of the second trimester as the uterus rises into a different position. b. the woman's increased urinary frequency in the first trimester is the result of exaggerated uterine anteflexion caused by softening. c. Braxton Hicks contractions become more painful in the third trimester, particularly if the woman tries to exercise. d. the uterine souffle is the movement of the fetus.

ANS: B The softening of the lower uterine segment is called Hegar's sign. Lightening occurs in the last 2 weeks of pregnancy, when the fetus descends. Braxton Hicks contractions become more defined in the final trimester but are not painful. Walking or exercise usually causes them to stop. The uterine souffle is the sound made by blood in the uterine arteries; it can be heard with a fetal stethoscope.

As relates to fetal positioning during labor, nurses should be aware that: a. position is a measure of the degree of descent of the presenting part of the fetus through the birth canal. b. birth is imminent when the presenting part is at +4 to +5 cm below the spine. c. the largest transverse diameter of the presenting part is the suboccipitobregmatic diameter. d. engagement is the term used to describe the beginning of labor.

ANS: B The station of the presenting part should be noted at the beginning of labor so that the rate of descent can be determined. Position is the relation of the presenting part of the fetus to the four quadrants of the mother's pelvis; station is the measure of degree of descent. The largest diameter usually is the biparietal diameter. The suboccipitobregmatic diameter is the smallest, although one of the most critical. Engagement often occurs in the weeks just before labor in nulliparas and before or during labor in multiparas.

Which statement about multifetal pregnancy is inaccurate? a. The expectant mother often develops anemia because the fetuses have a greater demand for iron. b. Twin pregnancies come to term with the same frequency as single pregnancies. c. The mother should be counseled to increase her nutritional intake and gain more weight. d. Backache and varicose veins often are more pronounced.

ANS: B Twin pregnancies often end in prematurity. Serious efforts should be made to bring the pregnancy to term. A woman with a multifetal pregnancy often develops anemia, suffers more or worse backache, and needs to gain more weight. Counseling is needed to help her adjust to these conditions.

Physiologic anemia often occurs during pregnancy as a result of: a. inadequate intake of iron. b. dilution of hemoglobin concentration. c. the fetus establishing iron stores. d. decreased production of erythrocytes.

ANS: B When blood volume expansion is more pronounced and occurs earlier than the increase in red blood cells, the woman has physiologic anemia, which is the result of dilution of hemoglobin concentration rather than inadequate hemoglobin. Inadequate intake of iron may lead to true anemia. There is an increased production of erythrocytes during pregnancy.

A pregnant woman at 25 weeks' gestation tells the nurse that she dropped a pan last week and her baby jumped at the noise. Which response by the nurse is most accurate? a. "That must have been a coincidence; babies can't respond like that." b. "The fetus is demonstrating the aural reflex." c. "Babies respond to sound starting at about 24 weeks of gestation." d. "Let me know if it happens again; we need to report that to your midwife."

ANS: C "Babies respond to sound starting at about 24 weeks of gestation" is an accurate statement. "That must have been a coincidence; babies can't respond like that" is inaccurate. Acoustic stimulations can evoke a fetal heart rate response. There is no such thing as an aural reflex. The statement, "Let me know if it happens again; we need to report that to your midwife" is not appropriate; it gives the impression that something is wrong.

The measurement of lecithin in relation to sphingomyelin (L/S ratio) is used to determine fetal lung maturity. Which ratio reflects maturity of the lungs? a. 1.4:1 b. 1.8:1 c. 2:1 d. 1:1

ANS: C A ratio of 2:1 indicates a two-to-one ratio of L/S, an indicator of lung maturity. Ratios of 1.4:1, 1.8:1, and 1:1 indicate immaturity of the fetal lungs.

With regard to the development of the respiratory system, maternity nurses should understand that: a. the respiratory system does not begin developing until after the embryonic stage. b. the infant's lungs are considered mature when the lecithin/sphingomyelin [L/S] ratio is 1:1, at about 32 weeks. c. maternal hypertension can reduce maternal-placental blood flow, accelerating lung maturity. d. fetal respiratory movements are not visible on ultrasound scans until at least 16 weeks.

ANS: C A reduction in placental blood flow stresses the fetus, increases blood levels of corticosteroids, and accelerates lung maturity. Development of the respiratory system begins during the embryonic phase and continues into childhood. The infant's lungs are mature when the L/S ratio is 2:1, at about 35 weeks. Lung movements have been seen on ultrasound scans at 11 weeks.

When assessing a woman in labor, the nurse is aware that the relationship of the fetal body parts to one another is called fetal: a. lie. b. presentation. c. attitude. d. position.

ANS: C Attitude is the relation of the fetal body parts to one another. Lie is the relation of the long axis (spine) of the fetus to the long axis (spine) of the mother. Presentation refers to the part of the fetus that enters the pelvic inlet first and leads through the birth canal during labor at term. Position is the relation of the presenting part to the four quadrants of the mother's pelvis.

With regard to the initial visit with a patient who is beginning prenatal care, nurses should be aware that: a. the first interview is a relaxed, get-acquainted affair in which nurses gather some general impressions. b. if nurses observe handicapping conditions, they should be sensitive and not enquire about them because the patient will do that in her own time. c. nurses should be alert to the appearance of potential parenting problems, such as depression or lack of family support. d. because of legal complications, nurses should not ask about illegal drug use; that is left to physicians.

ANS: C Besides these potential problems, nurses need to be alert to the woman's attitude toward health care. The initial interview needs to be planned, purposeful, and focused on specific content. A lot of ground must be covered. Nurses must be sensitive to special problems, but they do need to inquire because discovering individual needs is important. People with chronic or handicapping conditions forget to mention them because they have adapted to them. Getting information on drug use is important and can be done confidentially. Actual testing for drug use requires the patient's consent.

Which nursing assessment indicates that a woman who is in second-stage labor is almost ready to give birth? a. The fetal head is felt at 0 station during vaginal examination. b. Bloody mucus discharge increases. c. The vulva bulges and encircles the fetal head. d. The membranes rupture during a contraction.

ANS: C During the active pushing (descent) phase, the woman has strong urges to bear down as the presenting part of the fetus descends and presses on the stretch receptors of the pelvic floor. The vulva stretches and begins to bulge encircling the fetal head. Birth of the head occurs when the station is +4. A 0 station indicates engagement. Bloody show occurs throughout the labor process and is not an indication of an imminent birth. Rupture of membranes can occur at any time during the labor process and does not indicate an imminent birth.

Some pregnant patients may complain of changes in their voice and impaired hearing. The nurse can tell these patients that these are common reactions to: a. a decreased estrogen level. b. displacement of the diaphragm, resulting in thoracic breathing. c. congestion and swelling, which occur because the upper respiratory tract has become more vascular. d. increased blood volume.

ANS: C Estrogen levels increase, causing the upper respiratory tract to become more vascular producing swelling and congestion in the nose and ears leading to voice changes and impaired hearing. The diaphragm is displaced, and the volume of blood is increased. However, the main concern is increased estrogen levels.

Which meal would provide the most absorbable iron? a. Toasted cheese sandwich, celery sticks, tomato slices, and a grape drink b. Oatmeal, whole wheat toast, jelly, and low-fat milk c. Black bean soup, wheat crackers, orange sections, and prunes d. Red beans and rice, cornbread, mixed greens, and decaffeinated tea

ANS: C Food sources that are rich in iron include liver, meats, whole grain or enriched breads and cereals, deep green leafy vegetables, legumes, and dried fruits. In addition, the vitamin C in orange sections aids absorption. Dairy products and tea are not sources of iron.

While providing care to a patient in active labor, the nurse should instruct the woman that: a. the supine position commonly used in the United States increases blood flow. b. the "all fours" position, on her hands and knees, is hard on her back. c. frequent changes in position will help relieve her fatigue and increase her comfort. d. in a sitting or squatting position, her abdominal muscles will have to work harder.

ANS: C Frequent position changes relieve fatigue, increase comfort, and improve circulation. Blood flow can be compromised in the supine position; any upright position benefits cardiac output. The "all fours" position is used to relieve backache in certain situations. In a sitting or squatting position, the abdominal muscles work in greater harmony with uterine contractions.

The nurse is providing genetic counseling for an expectant couple who already have a child with trisomy 18. The nurse should: a. tell the couple they need to have an abortion within 2 to 3 weeks. b. explain that the fetus has a 50% chance of having the disorder. c. discuss options with the couple, including amniocentesis to determine whether the fetus is affected. d. refer the couple to a psychologist for emotional support.

ANS: C Genetic testing, including amniocentesis, would need to be performed to determine whether the fetus is affected. The couple should be given information about the likelihood of having another baby with this disorder so that they can make an informed decision. A genetic counselor is the best source for determining genetic probability ratios. The couple eventually may need emotional support, but the status of the pregnancy must be determined first.

Which minerals and vitamins usually are recommended to supplement a pregnant woman's diet? a. Fat-soluble vitamins A and D b. Water-soluble vitamins C and B6 c. Iron and folate d. Calcium and zinc

ANS: C Iron generally should be supplemented, and folic acid supplements often are needed because folate is so important. Fat-soluble vitamins should be supplemented as a medical prescription, as vitamin D might be for lactose-intolerant women. Water-soluble vitamin C sometimes is consumed in excess naturally; vitamin B6 is prescribed only if the woman has a very poor diet. Zinc sometimes is supplemented. Most women obtain enough calcium through their regular diet.

To reassure and educate pregnant patients about changes in their breasts, nurses should be aware that: a. the visibility of blood vessels that form an intertwining blue network indicates full function of Montgomery's tubercles and possibly infection of the tubercles. b. the mammary glands do not develop until 2 weeks before labor. c. lactation is inhibited until the estrogen level declines after birth. d. colostrum is the yellowish oily substance used to lubricate the nipples for breastfeeding.

ANS: C Lactation is inhibited until after birth. The visible blue network of blood vessels is a normal outgrowth of a richer blood supply. The mammary glands are functionally complete by midpregnancy. Colostrum is a creamy, white-to-yellow premilk fluid that can be expressed from the nipples before birth.

With regard to a woman's reordering of personal relationships during pregnancy, the maternity nurse should understand that: a. because of the special motherhood bond, a woman's relationship with her mother is even more important than with the father of the child. b. nurses need not get involved in any sexual issues the couple has during pregnancy, particularly if they have trouble communicating them to each other. c. women usually express two major relationship needs during pregnancy: feeling loved and valued and having the child accepted by the father. d. the woman's sexual desire is likely to be highest in the first trimester because of the excitement and because intercourse is physically easier.

ANS: C Love and support help a woman feel better about her pregnancy. The most important person to the pregnant woman is usually the father. Nurses can facilitate communication between partners about sexual matters if, as is common, they are nervous about expressing their worries and feelings. The second trimester is the time when a woman's sense of well-being, along with certain physical changes, increases her desire for sex. Desire is decreased in the first and third trimesters.

In practical terms regarding genetic health care, nurses should be aware that: a. genetic disorders affect people of all socioeconomic backgrounds, races, and ethnic groups equally. b. genetic health care is more concerned with populations than individuals. c. the most important of all nursing functions is providing emotional support to the family during counseling. d. taking genetic histories is the province of large universities and medical centers.

ANS: C Nurses should be prepared to help with various stress reactions from a couple facing the possibility of a genetic disorder. Although anyone may have a genetic disorder, certain disorders appear more often in certain ethnic and racial groups. Genetic health care is highly individualized because treatments are based on the phenotypic responses of the individual. Individual nurses at any facility can take a genetic history, although larger facilities may have better support services.

A patient at 24 weeks of gestation contacts the nurse at her obstetric provider's office to complain that she has cravings for dirt and gravel. The nurse is aware that this condition is known as ________ and may indicate anemia. a. ptyalism b. pyrosis c. pica d. decreased peristalsis

ANS: C Pica (a desire to eat nonfood substances) is an indication of iron deficiency and should be evaluated. Ptyalism (excessive salivation), pyrosis (heartburn), and decreased peristalsis are normal findings of gastrointestinal change during pregnancy. Food cravings during pregnancy are normal.

A woman's cousin gave birth to an infant with a congenital heart anomaly. The woman asks the nurse when such anomalies occur during development. Which response by the nurse is most accurate? a. "We don't really know when such defects occur." b. "It depends on what caused the defect." c. "They occur between the third and fifth weeks of development." d. "They usually occur in the first 2 weeks of development."

ANS: C The cardiovascular system is the first organ system to function in the developing human. Blood vessel and blood formation begins in the third week, and the heart is developmentally complete in the fifth week. "We don't really know when such defects occur" is an inaccurate statement. Regardless of the cause, the heart is vulnerable during its period of development, the third to fifth weeks. "They usually occur in the first 2 weeks of development" is an inaccurate statement.

A father and mother are carriers of phenylketonuria (PKU). Their 2-year-old daughter has PKU. The couple tells the nurse that they are planning to have a second baby. Because their daughter has PKU, they are sure that their next baby won't be affected. What response by the nurse is most accurate? a. "Good planning; you need to take advantage of the odds in your favor." b. "I think you'd better check with your doctor first." c. "You are both carriers, so each baby has a 25% chance of being affected." d. "The ultrasound indicates a boy, and boys are not affected by PKU."

ANS: C The chance is one in four that each child produced by this couple will be affected by PKU disorder. This couple still has an increased likelihood of having a child with PKU. Having one child already with PKU does not guarantee that they will not have another. These parents need to discuss their options with their physician. However, an opportune time has presented itself for the couple to receive correct teaching about inherited genetic risks. No correlation exists between gender and inheritance of the disorder because PKU is an autosomal recessive disorder.

While taking a diet history, the nurse might be told that the expectant mother has cravings for ice chips, cornstarch, and baking soda. This represents a nutritional problem known as: a. preeclampsia. b. pyrosis. c. pica. d. purging.

ANS: C The consumption of foods low in nutritional value or of nonfood substances (e.g., dirt, laundry starch) is called pica.

When assessing the fetus using Leopold maneuvers, the nurse feels a round, firm, movable fetal part in the fundal portion of the uterus and a long, smooth surface in the mother's right side close to midline. What is the likely position of the fetus? a. ROA b. LSP c. RSA d. LOA

ANS: C The fetus is positioned anteriorly in the right side of the maternal pelvis with the sacrum as the presenting part. RSA is the correct three-letter abbreviation to indicate this fetal position. The first letter indicates the presenting part in either the right or left side of the maternal pelvis. The second letter indicates the anatomic presenting part of the fetus. The third letter stands for the location of the presenting part in relation to the anterior, posterior, or transverse portion of the maternal pelvis. Palpation of a round, firm fetal part in the fundal portion of the uterus would be the fetal head, indicating that the fetus is in a breech position with the sacrum as the presenting part in the maternal pelvis. Palpation of the fetal spine along the mother's right side denotes the location of the presenting part in the mother's pelvis. The ability to palpate the fetal spine indicates that the fetus is anteriorly positioned in the maternal pelvis.

With regard to factors that affect how the fetus moves through the birth canal, nurses should be aware that: a. the fetal attitude describes the angle at which the fetus exits the uterus. b. of the two primary fetal lies, the horizontal lie is that in which the long axis of the fetus is parallel to the long axis of the mother. c. the normal attitude of the fetus is called general flexion. d. the transverse lie is preferred for vaginal birth.

ANS: C The normal attitude of the fetus is general flexion. The fetal attitude is the relation of fetal body parts to one another. The horizontal lie is perpendicular to the mother; in the longitudinal (or vertical) lie the long axes of the fetus and the mother are parallel. Vaginal birth cannot occur if the fetus stays in a transverse lie.

The nurse should be aware that the partner's main role in pregnancy is to: a. provide financial support. b. protect the pregnant woman from "old wives' tales." c. support and nurture the pregnant woman. d. make sure the pregnant woman keeps prenatal appointments.

ANS: C The partner's main role in pregnancy is to nurture the pregnant woman and to respond her feelings of vulnerability. In older societies, the man enacted the ritual couvade. Changing cultural and professional attitudes have encouraged fathers' participation in the birth experience over the past 30 years.

Sally comes in for her first prenatal examination. This is her first child. She asks you (the nurse), "How does my baby get air inside my uterus?" The correct response is: a. "The baby's lungs work in utero to exchange oxygen and carbon dioxide." b. "The baby absorbs oxygen from your blood system." c. "The placenta provides oxygen to the baby and excretes carbon dioxide into your bloodstream." d. "The placenta delivers oxygen-rich blood through the umbilical artery to the baby's abdomen."

ANS: C The placenta functions by supplying oxygen and excreting carbon dioxide to the maternal bloodstream. The fetal lungs do not function for respiratory gas exchange in utero. The baby does not simply absorb oxygen from a woman's blood system. Blood and gas transport occur through the placenta. The placenta delivers oxygen-rich blood through the umbilical vein and not the artery.

When providing care to the prenatal patient, the nurse understands that pica is defined as: a. intolerance of milk products. b. iron deficiency anemia. c. ingestion of nonfood substances. d. episodes of anorexia and vomiting.

ANS: C The practice of eating substances not normally thought of as food is called pica. Clay or dirt and solid laundry starch are the substances most commonly ingested. Intolerance of milk products is referred to as lactose intolerance. Pica may produce iron deficiency anemia if proper nutrition is decreased. Pica is not related to anorexia and vomiting.

To provide the patient with accurate information about dental care during pregnancy, maternity nurses should be aware that: a. dental care can be dropped from the priority list because the woman has enough to worry about and is getting a lot of calcium anyway. b. dental surgery, in particular, is contraindicated because of the psychologic stress it engenders. c. if dental treatment is necessary, the woman will be most comfortable with it in the second trimester. d. dental care interferes with the expectant mother's need to practice conscious relaxation.

ANS: C The second trimester is best for dental treatment because that is when the woman will be able to sit most comfortably in the dental chair. Dental care such as brushing with fluoride toothpaste is especially important during pregnancy because nausea during pregnancy may lead to poor oral hygiene. Emergency dental surgery is permissible, but the mother must clearly understand the risks and benefits. Conscious relaxation is useful, and it may even help the woman get through any dental appointments; it is not a reason to avoid them.

With regard to the turns and other adjustments of the fetus during the birth process, known as the mechanism of labor, nurses should be aware that: a. the seven critical movements must progress in a more or less orderly sequence. b. asynclitism sometimes is achieved by means of the Leopold maneuver. c. the effects of the forces determining descent are modified by the shape of the woman's pelvis and the size of the fetal head. d. at birth the baby is said to achieve "restitution" (i.e., a return to the C-shape of the womb).

ANS: C The size of the maternal pelvis and the ability of the fetal head to mold also affect the process. The seven identifiable movements of the mechanism of labor occur in combinations simultaneously, not in precise sequences. Asynclitism is the deflection of the baby's head; the Leopold maneuver is a means of judging descent by palpating the mother's abdomen. Restitution is the rotation of the baby's head after the infant is born.

With regard to follow-up visits for women receiving prenatal care, nurses should be aware that: a. the interview portions become more intensive as the visits become more frequent over the course of the pregnancy. b. monthly visits are scheduled for the first trimester, every 2 weeks for the second trimester, and weekly for the third trimester. c. during the abdominal examination, the nurse should be alert for supine hypotension. d. for pregnant women, a systolic blood pressure (BP) of 130 and a diastolic BP of 80 is sufficient to be considered hypertensive.

ANS: C The woman lies on her back during the abdominal examination, possibly compressing the vena cava and aorta, which can cause a decrease in blood pressure and a feeling of faintness. The interview portion of follow-up examinations is less extensive than in the initial prenatal visits, during which so much new information must be gathered. Monthly visits are routinely scheduled for the first and second trimesters; visits increase to every 2 weeks at week 28 and to once a week at week 36. For pregnant women hypertension is defined as a systolic BP of 140 or greater and a diastolic BP of 90 or greater.

At approximately _____ weeks of gestation, lecithin is forming on the alveolar surfaces, the eyelids open, and the fetus measures approximately 27 cm crown to rump and weighs approximately 1110 g. a. 20 b. 24 c. 28 d. 30

ANS: C These milestones human development occur at approximately 28 weeks.

A pregnant woman's diet consists almost entirely of whole grain breads and cereals, fruits, and vegetables. The nurse would be most concerned about this woman's intake of: a. calcium. b. protein. c. vitamin B12. d. folic acid.

ANS: C This diet is consistent with that followed by a strict vegetarian (vegan). Vegans consume only plant products. Because vitamin B12 is found in foods of animal origin, this diet is deficient in vitamin B12.

A 27-year-old pregnant woman had a preconceptual body mass index (BMI) of 18.0. The nurse knows that this woman's total recommended weight gain during pregnancy should be at least: a. 20 kg (44 lbs). b. 16 kg (35 lbs). c. 12.5 kg (27.5 lbs). d. 10 kg (22 lbs).

ANS: C This woman has a normal BMI and should gain 11.5 to 16 kg during pregnancy. A weight gain of 20 kg would be unhealthy for most women. A weight gain 35 lbs is the high end of the range of weight this woman should gain in her pregnancy. A weight gain of 22 lbs would be appropriate for an obese woman.

A pregnant woman at 10 weeks of gestation jogs three or four times per week. She is concerned about the effect of exercise on the fetus. The nurse should inform her: a. "You don't need to modify your exercising any time during your pregnancy." b. "Stop exercising because it will harm the fetus." c. "You may find that you need to modify your exercise to walking later in your pregnancy, around the seventh month." d. "Jogging is too hard on your joints; switch to walking now."

ANS: C Typically running should be replaced with walking around the seventh month of pregnancy. The nurse should inform the woman that she may need to reduce her exercise level as the pregnancy progresses. Physical activity promotes a feeling of well-being in pregnant women. It improves circulation, promotes relaxation and rest, and counteracts boredom. Simple measures should be initiated to prevent injuries, such as warm-up and stretching exercises to prepare the joints for more strenuous exercise.

The maternity nurse understands that as the uterus contracts during labor, maternal-fetal exchange of oxygen and waste products: a. continues except when placental functions are reduced. b. increases as blood pressure decreases. c. diminishes as the spiral arteries are compressed. d. is not significantly affected.

ANS: C Uterine contractions during labor tend to decrease circulation through the spiral electrodes and subsequent perfusion through the intervillous space. The maternal blood supply to the placenta gradually stops with contractions. The exchange of oxygen and waste products decreases. The exchange of oxygen and waste products is affected by contractions.

During pregnancy, many changes occur as a direct result of the presence of the fetus. Which of these adaptations meet this criterion? (Select all that apply.) a. Leukorrhea b. Development of the operculum c. Quickening d. Ballottement e. Lightening

ANS: C, D, E Leukorrhea is a white or slightly gray vaginal discharge that develops in response to cervical stimulation by estrogen and progesterone. Quickening is the first recognition of fetal movements or "feeling life." Quickening is often described as a flutter and is felt earlier in multiparous women than in primiparas. Lightening occurs when the fetus begins to descend into the pelvis. This occurs 2 weeks before labor in the nullipara and at the start of labor in the multipara. Mucus fills the cervical canal creating a plug otherwise known as the operculum. The operculum acts as a barrier against bacterial invasion during the pregnancy. Passive movement of the unengaged fetus is referred to as ballottement.

A woman has come to the clinic for preconception counseling because she wants to start trying to get pregnant in 3 months. She can expect the following advice: a. "Discontinue all contraception now." b. "Lose weight so that you can gain more during pregnancy." c. "You may take any medications you have been taking regularly." d. "Make sure that you include adequate folic acid in your diet."

ANS: D A healthy diet before conception is the best way to ensure that adequate nutrients are available for the developing fetus. A woman's folate or folic acid intake is of particular concern in the periconception period. Neural tube defects are more common in infants of women with a poor folic acid intake. Depending on the type of contraception used, discontinuing all contraception may not be appropriate advice. Losing weight is not appropriate advice. Depending on the type of medication the woman is taking, continuing its use may not be appropriate.

Which basic type of pelvis includes the correct description and percentage of occurrence in women? a. Gynecoid: classic female; heart shaped; 75% b. Android: resembling the male; wider oval; 15% c. Anthropoid: resembling the ape; narrower; 10% d. Platypelloid: flattened, wide, shallow; 3%

ANS: D A platypelloid pelvis is flattened, wide, and shallow; about 3% of women have this shape. The gynecoid shape is the classical female shape, slightly ovoid and rounded; about 50% of women have this shape. An android, or male-like, pelvis is heart shaped; about 23% of women have this shape. An anthropoid, or ape-like, pelvis is oval and wider; about 24% of women have this shape.

Which pregnant woman should restrict her weight gain during pregnancy? a. Woman pregnant with twins b. Woman in early adolescence c. Woman shorter than 62 inches or 157 cm d. Woman who was 20 lbs overweight before pregnancy

ANS: D A weight gain of 5 to 9 kg will provide sufficient nutrients for the fetus. Overweight and obese women should be advised to lose weight before conception to achieve the best pregnancy outcomes. A higher weight gain in twin gestations may help prevent low birth weights. Adolescents need to gain weight toward the higher acceptable range, which provides for their own growth as well as for fetal growth. In the past, women of short stature were advised to restrict their weight gain; however, evidence to support these guidelines has not been found.

The most basic information a maternity nurse should have concerning conception is that: a. ova are considered fertile 48 to 72 hours after ovulation. b. sperm remain viable in the woman's reproductive system for an average of 12 to 24 hours. c. conception is achieved when a sperm successfully penetrates the membrane surrounding the ovum. d. implantation in the endometrium occurs 6 to 10 days after conception.

ANS: D After implantation, the endometrium is called the decidua. Ova are considered fertile for about 24 hours after ovulation. Sperm remain viable in the woman's reproductive system for an average of 2 to 3 days. Penetration of the ovum by the sperm is called fertilization. Conception occurs when the zygote, the first cell of the new individual, is formed.

The phenomenon of someone other than the mother-to-be experiencing pregnancy-like symptoms such as nausea and weight gain applies to the: a. mother of the pregnant woman. b. couple's teenage daughter. c. sister of the pregnant woman. d. expectant father.

ANS: D An expectant father's experiencing pregnancy-like symptoms is called the couvade syndrome

While you are assessing the vital signs of a pregnant woman in her third trimester, the patient complains of feeling faint, dizzy, and agitated. Which nursing intervention is appropriate? a. Have the patient stand up and retake her blood pressure. b. Have the patient sit down and hold her arm in a dependent position. c. Have the patient lie supine for 5 minutes and recheck her blood pressure on both arms. d. Have the patient turn to her left side and recheck her blood pressure in 5 minutes.

ANS: D Blood pressure is affected by maternal position during pregnancy. The supine position may cause occlusion of the vena cava and descending aorta. Turning the pregnant woman to a lateral recumbent position alleviates pressure on the blood vessels and quickly corrects supine hypotension. Pressures are significantly higher when the patient is standing. This option causes an increase in systolic and diastolic pressures. The arm should be supported at the same level of the heart. The supine position may cause occlusion of the vena cava and descending aorta, creating hypotension.

The _____ is/are responsible for oxygen and carbon dioxide transport to and from the maternal bloodstream. a. decidua basalis b. blastocyst c. germ layer d. chorionic villi

ANS: D Chorionic villi are finger-like projections that develop out of the trophoblast and extend into the blood-filled spaces of the endometrium. The villi obtain oxygen and nutrients from the maternal bloodstream and dispose of carbon dioxide and waste products into the maternal blood. The decidua basalis is the portion of the decidua (endometrium) under the blastocyst where the villi attach. The blastocyst is the embryonic development stage after the morula. Implantation occurs at this stage. The germ layer is a layer of the blastocyst.

The major source of nutrients in the diet of a pregnant woman should be composed of: a. simple sugars. b. fats. c. fiber. d. complex carbohydrates.

ANS: D Complex carbohydrates supply the pregnant woman with vitamins, minerals, and fiber. The most common simple carbohydrate is table sugar, which is a source of energy but does not provide any nutrients. Fats provide 9 kcal in each gram, in contrast to carbohydrates and proteins, which provide only 4 kcal in each gram. Fiber is supplied primarily by complex carbohydrates.

To reassure and educate their pregnant patients about changes in their blood pressure, maternity nurses should be aware that: a. a blood pressure cuff that is too small produces a reading that is too low; a cuff that is too large produces a reading that is too high. b. shifting the patient's position and changing from arm to arm for different measurements produces the most accurate composite blood pressure reading at each visit. c. the systolic blood pressure increases slightly as pregnancy advances; the diastolic pressure remains constant. d. compression of the iliac veins and inferior vena cava by the uterus contributes to hemorrhoids in the later stage of term pregnancy.

ANS: D Compression of the iliac veins and inferior vena cava also leads to varicose veins in the legs and vulva. The tightness of a cuff that is too small produces a reading that is too high; similarly the looseness of a cuff that is too large results in a reading that is too low. Because maternal positioning affects readings, blood pressure measurements should be obtained in the same arm and with the woman in the same position. The systolic blood pressure generally remains constant but may decline slightly as pregnancy advances. The diastolic blood pressure first decreases and then gradually increases.

When counseling a patient about getting enough iron in her diet, the maternity nurse should tell her that: a. milk, coffee, and tea aid iron absorption if consumed at the same time as iron. b. iron absorption is inhibited by a diet rich in vitamin C. c. iron supplements are permissible for children in small doses. d. constipation is common with iron supplements.

ANS: D Constipation can be a problem. Milk, coffee, and tea inhibit iron absorption when consumed at the same time as iron. Vitamin C promotes iron absorption. Children who ingest iron can get very sick and even die.

A woman who is 8 months pregnant asks the nurse, "Does my baby have any antibodies to fight infection?" The most appropriate response by the nurse is: a. "Your baby has all the immune globulins necessary: IgG, IgM, and IgA." b. "Your baby won't receive any antibodies until he/she is born and you breastfeed him." c. "Your baby does not have any antibodies to fight infection." d. "Your baby has IgG and IgM."

ANS: D During the third trimester, the only immune globulin that crosses the placenta, IgG, provides passive acquired immunity to specific bacterial toxins. The fetus produces IgM by the end of the first trimester. IgA is not produced by the baby. By the third trimester, the fetus has IgG and IgM. Breastfeeding supplies the baby with IgA. "Your baby does not have any antibodies to fight infection" is an inaccurate statement.

With regard to the estimation and interpretation of the recurrence of risks for genetic disorders, nurses should be aware that: a. with a dominant disorder, the likelihood of the second child also having the condition is 100%. b. an autosomal recessive disease carries a one in eight risk of the second child also having the disorder. c. disorders involving maternal ingestion of drugs carry a one in four chance of being repeated in the second child. d. the risk factor remains the same no matter how many affected children are already in the family.

ANS: D Each pregnancy is an independent event. The risk factor (e.g., one in two, one in four) remains the same for each child, no matter how many children are born to the family. In a dominant disorder, the likelihood of recurrence in subsequent children is 50% (one in two). An autosomal recessive disease carries a one in four chance of recurrence. In disorders involving maternal ingestion of drugs, subsequent children would be at risk only if the mother continued to take drugs; the rate of risk would be difficult to calculate.

A woman in week 34 of pregnancy reports that she is very uncomfortable because of heartburn. The nurse would suggest that the woman: a. substitute other calcium sources for milk in her diet. b. lie down after each meal. c. reduce the amount of fiber she consumes. d. eat five small meals daily.

ANS: D Eating small, frequent meals may help with heartburn, nausea, and vomiting. Substituting other calcium sources for milk, lying down after eating, and reducing fiber intake are inappropriate dietary suggestions for all pregnant women and do not alleviate heartburn.

In order to care for obstetric patients adequately, the nurse understands that labor contractions facilitate cervical dilation by: a. contracting the lower uterine segment. b. enlarging the internal size of the uterus. c. promoting blood flow to the cervix. d. pulling the cervix over the fetus and amniotic sac.

ANS: D Effective uterine contractions pull the cervix upward at the same time that the fetus and amniotic sac are pushed downward. The contractions are stronger at the fundus. The internal size becomes smaller with the contractions; this helps to push the fetus down. Blood flow decreases to the uterus during a contraction.

Human chorionic gonadotropin (hCG) is an important biochemical marker for pregnancy and the basis for many tests. A maternity nurse should be aware that: a. hCG can be detected 2.5 weeks after conception. b. the hCG level increases gradually and uniformly throughout pregnancy. c. much lower than normal increases in the level of hCG may indicate a postdate pregnancy. d. a higher than normal level of hCG may indicate an ectopic pregnancy or Down syndrome.

ANS: D Higher levels also could be a sign of multiple gestation. hCG can be detected 7 to 8 days after conception. The hCG level fluctuates during pregnancy: peaking, declining, stabilizing, and increasing again. Abnormally slow increases may indicate impending miscarriage.

To reassure and educate pregnant patients about changes in the cervix, vagina, and position of the fetus, nurses should be aware that: a. because of a number of changes in the cervix, abnormal Papanicolaou (Pap) tests are much easier to evaluate. b. Quickening is a technique of palpating the fetus to engage it in passive movement. c. the deepening color of the vaginal mucosa and cervix (Chadwick's sign) usually appears in the second trimester or later as the vagina prepares to stretch during labor. d. increased vascularity of the vagina increases sensitivity and may lead to a high degree of arousal, especially in the second trimester.

ANS: D Increased sensitivity and an increased interest in sex sometimes go together. This frequently occurs during the second trimester. Cervical changes make evaluation of abnormal Pap tests more difficult. Quickening is the first recognition of fetal movements by the mother. Ballottement is a technique used to palpate the fetus. Chadwick's sign appears from the sixth to eighth weeks.

Which statement about a condition of pregnancy is accurate? a. Insufficient salivation (ptyalism) is caused by increases in estrogen. b. Acid indigestion (pyrosis) begins early but declines throughout pregnancy. c. Hyperthyroidism often develops (temporarily) because hormone production increases. d. Nausea and vomiting rarely have harmful effects on the fetus and may be beneficial.

ANS: D Normal nausea and vomiting rarely produce harmful effects, and nausea and vomiting periods may be less likely to result in miscarriage or preterm labor. Ptyalism is excessive salivation, which may be caused by a decrease in unconscious swallowing or stimulation of the salivary glands. Pyrosis begins in the first trimester and intensifies through the third trimester. Increased hormone production does not lead to hyperthyroidism in pregnant women.

When discussing work and travel during pregnancy with a pregnant patient, nurses should instruct them that: a. women should sit for as long as possible and cross their legs at the knees from time to time for exercise. b. women should avoid seat belts and shoulder restraints in the car because they press on the fetus. c. metal detectors at airport security checkpoints can harm the fetus if the woman passes through them a number of times. d. while working or traveling in a car or on a plane, women should arrange to walk around at least every 2 hours or so.

ANS: D Periodic walking helps prevent thrombophlebitis. Pregnant women should avoid sitting or standing for long periods and crossing the legs at the knees. Pregnant women must wear lap belts and shoulder restraints. The most common injury to the fetus comes from injury to the mother. Metal detectors at airport security checkpoints do not harm fetuses.

The nurse caring for the pregnant patient must understand that the hormone essential for maintaining pregnancy is: a. estrogen. b. human chorionic gonadotropin (hCG). c. oxytocin. d. progesterone.

ANS: D Progesterone is essential for maintaining pregnancy; it does so by relaxing smooth muscles. This reduces uterine activity and prevents miscarriage. Estrogen plays a vital role in pregnancy, but it is not the primary hormone for maintaining pregnancy. hCG levels increase at implantation but decline after 60 to 70 days. Oxytocin stimulates uterine contractions.

A pregnant woman's diet history indicates that she likes the following list of foods. The nurse would encourage this woman to consume more of which food to increase her calcium intake? a. Fresh apricots b. Canned clams c. Spaghetti with meat sauce d. Canned sardines

ANS: D Sardines are rich in calcium. Fresh apricots, canned clams, and spaghetti with meat sauce are not high in calcium.

Which symptom is considered a warning sign and should be reported immediately by the pregnant woman to her health care provider? a. Nausea with occasional vomiting b. Fatigue c. Urinary frequency d. Vaginal bleeding

ANS: D Signs and symptoms that must be reported include severe vomiting, fever and chills, burning on urination, diarrhea, abdominal cramping, and vaginal bleeding. These symptoms may be signs of potential complications of the pregnancy. Nausea with occasional vomiting, fatigue, and urinary frequency are normal first-trimester complaints. Although they may be worrisome or annoying to the mother, they usually are not indications of pregnancy problems.

A pregnant woman's diet may not meet her need for folates. A good source of this nutrient is: a. chicken. b. cheese. c. potatoes. d. green leafy vegetables.

ANS: D Sources of folates include green leafy vegetables, whole grains, fruits, liver, dried peas, and beans. Chicken and cheese are excellent sources of protein but are poor in folates. Potatoes contain carbohydrates and vitamins and minerals but are poor in folates.

Numerous changes in the integumentary system occur during pregnancy. Which change persists after birth? a. Epulis b. Chloasma c. Telangiectasia d. Striae gravidarum

ANS: D Striae gravidarum, or stretch marks, reflect separation within the underlying connective tissue of the skin. They usually fade after birth, although they never disappear completely. An epulis is a red, raised nodule on the gums that bleeds easily. Chloasma, or mask of pregnancy, is a blotchy, brown hyperpigmentation of the skin over the cheeks, nose, and forehead, especially in dark-complexioned pregnant women. Chloasma usually fades after the birth. Telangiectasia, or vascular spiders, are tiny, star-shaped or branch-like, slightly raised, pulsating end-arterioles usually found on the neck, thorax, face, and arms. They occur as a result of elevated levels of circulating estrogen. These usually disappear after birth.

In order to evaluate the condition of the patient accurately during labor, the nurse should be aware that: a. the woman's blood pressure will increase during contractions and fall back to prelabor normal between contractions. b. use of the Valsalva maneuver is encouraged during the second stage of labor to relieve fetal hypoxia. c. having the woman point her toes will reduce leg cramps. d. the endogenous endorphins released during labor will raise the woman's pain threshold and produce sedation.

ANS: D The endogenous endorphins released during labor will raise the woman's pain threshold and produce sedation. In addition, physiologic anesthesia of the perineal tissues, caused by the pressure of the presenting part, decreases the mother's perception of pain. Blood pressure increases during contractions but remains somewhat elevated between them. Use of the Valsalva maneuver is discouraged during second-stage labor because of a number of unhealthy outcomes, including fetal hypoxia. Pointing the toes can cause leg cramps, as can the process of labor itself.

The factors that affect the process of labor and birth, known commonly as the five Ps, include all except: a. passenger. b. passageway. c. powers. d. pressure.

ANS: D The five Ps are passenger (fetus and placenta), passageway (birth canal), powers (contractions), position of the mother, and psychologic response.

A new mother asks the nurse when the "soft spot" on her son's head will go away. The nurse's answer is based on the knowledge that the anterior fontanel closes after birth by _____ months. a. 2 b. 8 c. 12 d. 18

ANS: D The larger of the two fontanels, the anterior fontanel, closes by 18 months after birth.

Nutrition is one of the most significant factors influencing the outcome of a pregnancy. It is an alterable and important preventive measure for various potential problems, such as low birth weight and prematurity. While completing the physical assessment of the pregnant patient, the nurse can evaluate the patient's nutritional status by observing a number of physical signs. Which sign would indicate that the patient has unmet nutritional needs? a. Normal heart rate, rhythm, and blood pressure b. Bright, clear, shiny eyes c. Alert, responsive, and good endurance d. Edema, tender calves, and tingling

ANS: D The physiologic changes of pregnancy may complicate the interpretation of physical findings. Lower-extremity edema often occurs when caloric and protein deficiencies are present; however, it may also be a common physical finding during the third trimester. It is essential that the nurse complete a thorough health history and physical assessment and request further laboratory testing if indicated. A malnourished pregnant patient may display rapid heart rate, abnormal rhythm, enlarged heart, and elevated blood pressure. A patient receiving adequate nutrition has bright, shiny eyes with no sores and moist, pink membranes. Pale or red membranes, dryness, infection, dull appearance of the cornea, or blue sclerae all are signs of poor nutrition. This patient is well nourished. Cachexia, listlessness, and tiring easily would be indications of poor nutritional status.

In her work with pregnant women of various cultures, a nurse practitioner has observed various practices that seemed strange or unusual. She has learned that cultural rituals and practices during pregnancy seem to have one purpose in common. Which statement best describes that purpose? a. To promote family unity b. To ward off the "evil eye" c. To appease the gods of fertility d. To protect the mother and fetus during pregnancy

ANS: D The purpose of all cultural practices is to protect the mother and fetus during pregnancy. Although many cultures consider pregnancy normal, certain practices are expected of women of all cultures to ensure a good outcome. Cultural prescriptions tell women what to do, and cultural proscriptions establish taboos. The purposes of these practices are to prevent maternal illness resulting from a pregnancy-induced imbalanced state and to protect the vulnerable fetus.

A woman who is 14 weeks pregnant tells the nurse that she always had a glass of wine with dinner before she became pregnant. She has abstained during her first trimester and would like to know if it is safe for her to have a drink with dinner now. The nurse would tell her: a. "Since you're in your second trimester, there's no problem with having one drink with dinner." b. "One drink every night is too much. One drink three times a week should be fine." c. "Since you're in your second trimester, you can drink as much as you like." d. "Because no one knows how much or how little alcohol it takes to cause fetal problems, the best course is to abstain throughout your pregnancy."

ANS: D The statement "Because no one knows how much or how little alcohol it takes to cause fetal problems, the best course is to abstain throughout your pregnancy" is accurate. A safe level of alcohol consumption during pregnancy has not yet been established. Although the consumption of occasional alcoholic beverages may not be harmful to the mother or her developing fetus, complete abstinence is strongly advised.

A woman is 3 months pregnant. At her prenatal visit, she tells the nurse that she does not know what is happening; one minute she's happy that she is pregnant, and the next minute she cries for no reason. Which response by the nurse is most appropriate? a. "Don't worry about it; you'll feel better in a month or so." b. "Have you talked to your husband about how you feel?" c. "Perhaps you really don't want to be pregnant." d. "Hormonal changes during pregnancy commonly result in mood swings."

ANS: D The statement "Hormonal changes during pregnancy commonly result in mood swings" is accurate and the most appropriate response by the nurse. The statement "Don't worry about it; you'll feel better in a month or so" dismisses the patient's concerns and is not the most appropriate response. Although women should be encouraged to share their feelings, "Have you talked to your husband about how you feel" is not the most appropriate response and does not provide the patient with a rationale for the psychosocial dynamics of her pregnancy. "Perhaps you really don't want to be pregnant" is completely inappropriate and deleterious to the psychologic well-being of the woman. Hormonal and metabolic adaptations often cause mood swings in pregnancy. The woman's responses are normal. She should be reassured about her feelings.

A 22-year-old woman pregnant with a single fetus has a preconception body mass index (BMI) of 24. When she was seen in the clinic at 14 weeks of gestation, she had gained 1.8 kg (4 lbs) since conception. How would the nurse interpret this? a. This weight gain indicates possible gestational hypertension. b. This weight gain indicates that the woman's infant is at risk for intrauterine growth restriction (IUGR). c. This weight gain cannot be evaluated until the woman has been observed for several more weeks. d. The woman's weight gain is appropriate for this stage of pregnancy.

ANS: D The statement "The woman's weight gain is appropriate for this stage of pregnancy" is accurate. This woman's BMI is within the normal range. During the first trimester, the average total weight gain is only 1 to 2 kg. Although weight gain does indicate possible gestational hypertension, it does not apply to this patient. The desirable weight gain during pregnancy varies among women. The primary factor to consider in making a weight gain recommendation is the appropriateness of the prepregnancy weight for the woman's height. A commonly used method of evaluating the appropriateness of weight for height is the BMI. Although weight gain does indicate risk for IUGR, this does not apply to this patient. Weight gain should occur at a steady rate throughout the pregnancy. The optimal rate of weight gain also depends on the stage of the pregnancy.

What represents a typical progression through the phases of a woman's establishing a relationship with the fetus? a. Accepts the fetus as distinct from herself—accepts the biologic fact of pregnancy—has a feeling of caring and responsibility. b. Fantasizes about the child's gender and personality—views the child as part of herself—becomes introspective. c. Views the child as part of herself—has feelings of well-being—accepts the biologic fact of pregnancy. d. "I am pregnant."—"I am going to have a baby."—"I am going to be a mother."

ANS: D The woman first centers on herself as pregnant, then on the baby as an entity separate from herself, and then on her responsibilities as a mother. The expressions, "I am pregnant," "I am going to have a baby," and "I am going to be a mother" sum up the progression through the three phases.

A primigravida at 39 weeks of gestation is observed for 2 hours in the intrapartum unit. The fetal heart rate has been normal. Contractions are 5 to 9 minutes apart, 20 to 30 seconds in duration, and of mild intensity. Cervical dilation is 1 to 2 cm and uneffaced (unchanged from admission). Membranes are intact. The nurse should expect the woman to be: a. admitted and prepared for a cesarean birth. b. admitted for extended observation. c. discharged home with a sedative. d. discharged home to await the onset of true labor.

ANS: D This situation describes a woman with normal assessments who is probably in false labor and will probably not deliver rapidly once true labor begins. These are all indications of false labor without fetal distress. There is no indication that further assessment or cesarean birth is indicated. The patient will likely be discharged; however, there is no indication that a sedative is needed.

To determine the cultural influence on a patient's diet, the nurse should first: a. evaluate the patient's weight gain during pregnancy. b. assess the socioeconomic status of the patient. c. discuss the four food groups with the patient. d. identify the food preferences and methods of food preparation common to that culture.

ANS: D Understanding the patient's food preferences and how she prepares food will assist the nurse in determining whether the patient's culture is adversely affecting her nutritional intake. Evaluation of a patient's weight gain during pregnancy should be included for all patients, not just for patients who are culturally different. The socioeconomic status of the patient may alter the nutritional intake but not the cultural influence. Teaching the food groups to the patient should come after assessing food preferences.

Which vitamins or minerals can lead to congenital malformations of the fetus if taken in excess by the mother? a. Zinc b. Vitamin D c. Folic acid d. Vitamin A

ANS: D Zinc, vitamin D, and folic acid are vital to good maternal and fetal health and are highly unlikely to be consumed in excess. Vitamin A taken in excess causes a number of problems. An analog of vitamin A appears in prescribed acne medications, which must not be taken during pregnancy.

Fetal jeopardy or intrauterine fetal death

Decreased fetal movement

Hypertension, preeclampsia

Epigastric pain in late pregnancy

In response to requests by the U.S. Public Health Service for new models of prenatal care, an innovative new approach to prenatal care known as centering pregnancy was developed. Which statement would accurately apply to the centering model of care? a. Group sessions begin with the first prenatal visit. b. At each visit, blood pressure, weight, and urine dipsticks are obtained by the nurse. c. Eight to twelve women are placed in gestational-age cohort groups. d. Outcomes are similar to those of traditional prenatal care.

Gestational-age cohorts comprise the groups with approximately 8 to 12 women in each group. This group remains intact throughout the pregnancy. Individual follow-up visits are scheduled as needed. Group sessions begin at 12 to 16 weeks of gestation and end with an early after birth visit. Before group sessions the patient has an individual assessment, physical examination, and history. At the beginning of each group meeting, patients measure their own blood pressure, weight, and urine dips and enter these in their record. Fetal heart rate assessment and fundal height are obtained by the nurse. Results evaluating this approach have been very promising. In a study of adolescent patients, there was a decrease in low-birth-weight infants and an increase in breastfeeding rates.

Gestational diabetes

Glycosuria

Kidney infection or stones

Severe backache and flank pain

Hyperemesis gravidarum

Severe vomiting in early pregnancy


Related study sets

Section II - Life Insurance Practice Exam

View Set

Diet and Nutrition Module 2 Chapter 7 Energy Metabolism

View Set

Chapter 1: Introduction to Sociology

View Set

American Literature Proctored Exam

View Set

Business Finance Final Exam (Chapter 5)

View Set

Abdomen/Lower Limb/Etc: Lev Quiz

View Set

Principles of Accounting Module 6

View Set

ATI Pharmacology Practice Set #4

View Set